Inductor y condensador con suministro de CC

Aquí hay un circuito LC con un suministro de CC.

Cuando el interruptor se cierra en t=0, el condensador se comporta como un cortocircuito, mientras que el inductor se comporta como un circuito abierto, ya que el voltaje en el inductor salta inmediatamente al voltaje de la batería.

Ahora sabemos que la corriente en el inductor aumenta mientras que en un condensador la corriente disminuye con respecto al tiempo.

Entonces, me pregunto cómo se comportará la corriente en general en ese circuito, ya que cuando el capacitor está en cortocircuito y el inductor está en circuito abierto en t = 0, entonces, cuando la corriente en el capacitor disminuye, al mismo tiempo en el inductor se espera que aumente. Entonces, ¿cuál será la forma general de la corriente con respecto al tiempo? ¿Cómo se verá su gráfico y cuál es la explicación teórica para eso sin entrar en las ecuaciones diferenciales?

ingrese la descripción de la imagen aquí

Se comportará como una resistencia.

Respuestas (4)

Suponiendo que no hay resistencia en el circuito, la corriente en el circuito estará dada por la ecuación I ( t ) = mi C L pecado ω 0 t dónde ω 0 = 1 L C .

ingrese la descripción de la imagen aquí

En todo momento en su circuito, el voltaje total debe sumar cero.

mi + v C a pag a C i t o r + v i norte d tu C t o r = 0 [ mi + q C + L d I d t = 0 ]

con la ecuación diferencial que no desea utilizar entre paréntesis.

Para tratar de explicar lo que sucede, he dibujado una serie de diagramas secuenciados en el tiempo con T = 2 π L C .

ingrese la descripción de la imagen aquí

Diagrama 1
En el instante en que se cierra el interruptor, la corriente es cero y el voltaje a través del inductor se opone al voltaje aplicado desde la celda porque, aunque la corriente es cero, hay una tasa de cambio de corriente.
No hay carga en el capacitor, por lo que el voltaje a través del capacitor es cero.

Diagrama 2
Ahora hay una corriente, i , en el circuito, el condensador se está cargando, lo cual he mostrado con los signos más y menos entre las "placas" del condensador y hay un voltaje. v C a través del condensador.
Sin embargo, la tasa de cambio de la corriente ha disminuido, por lo que ahora hay un voltaje menor en el inductor v_{\rm L}$.
Como hay una corriente que fluye a través del inductor, tiene energía almacenada en su campo magnético (que se muestra en rojo) y también hay energía almacenada en el campo eléctrico producido por el capacitor.
Toda esa energía ha venido de la célula.

Diagrama 3
La corriente en el circuito alcanza un valor máximo I y el voltaje a través del capacitor ahora es igual en magnitud al voltaje a través de la celda mi .
En este momento, la tasa instantánea de cambio de corriente es cero, por lo que no hay voltaje en el inductor y, nuevamente, el voltaje total en el circuito es cero.
Tanto el inductor como el capacitor tienen más energía almacenada en sus campos.

Diagrama 4
Este diagrama puede sorprenderlo porque el voltaje en el capacitor ahora es mayor que el voltaje en la celda.
Esto sucede porque la corriente que se muestra fluyendo en el Diagrama 3 no puede dejar de fluir instantáneamente, por lo que el capacitor continúa cargándose pero con una corriente reducida. i en el circuito
Tenga en cuenta que debido a que la corriente ahora está disminuyendo, el voltaje a través del inductor ha invertido la polaridad y nuevamente el voltaje total en el circuito es cero.
El inductor ha cedido parte de su energía almacenada, pero la energía almacenada en el condensador sigue aumentando.

Diagrama 5
Eventualmente, el voltaje a través del capacitor alcanza el doble del voltaje de la celda y la corriente deja de fluir.
La carga en el capacitor es máxima y también lo es la energía almacenada dentro de él.
Aunque no fluye corriente a través del inductor, todavía hay una tasa instantánea de cambio de corriente que produce un voltaje a través del inductor igual al voltaje de la celda. mi y entonces el voltaje total en el circuito sigue siendo cero.
El inductor no tiene un campo magnético asociado y, por lo tanto, no almacena energía.

Con suerte, ahora podrá seguir los diagramas posteriores y darse cuenta de que el diagrama después del Diagrama 8 es el Diagrama 1 , ya que toda la secuencia se repite (para siempre).

En general, en un ciclo no hay transferencia de energía neta entre la celda y el resto del circuito.


El voltaje a través del capacitor es v C = ( ) mi ( 1 porque ω 0 t ) y el voltaje a través del inductor es v L = ( ) mi porque ω 0 t .


Si hubiera habido resistencia en el circuito entonces la corriente tendería a cero como el tiempo tendería a infinito con la forma exacta de la variación de la corriente con el tiempo dependiendo de los valores de la capacitancia, inductancia y resistencia en el circuito.


Observará que hay una mayor similitud entre su circuito y el circuito que se trató en esta pregunta donde el capacitor tenía una carga inicial y no había celda en el circuito.


Actualización como resultado de un comentario de @Alex

Para ilustrar lo que sucede a medida que cambia la resistencia, he usado una copia de Circuit Sandbox de MIT para producir algunos gráficos de corriente contra tiempo para un circuito LCR en serie con una entrada de paso y diferentes valores de resistencia.
(Tenga en cuenta que este Circuit Sandbox no funcionó para mí usando Firefox, así que usé Edge en su lugar).

ingrese la descripción de la imagen aquí

El amortiguamiento crítico para este circuito ocurre cuando la resistencia es 2 Ω y el sistema alcanza su estado estable (corriente = cero) sin sobreimpulso en el menor tiempo posible.

—-

Aquí están los gráficos de voltaje y corriente cuando la resistencia es 0.2 Ω .

ingrese la descripción de la imagen aquí

Sugerencia pedante: ω suele ser una variable mientras que ω 0 es un parámetro, por ejemplo, ω 0 = 1 L C
@AlfredCentauri Muchas gracias por su sugerencia, en la que he actuado. No habría tenido ninguna objeción a que hicieras tal cambio. :-)
realmente muchas gracias @Farcher. gran respuesta. Ahora, ¿qué sucede si agregamos una resistencia en serie con el inductor y el capacitor? ¿Cómo cambiará la resistencia la forma de la corriente? después de resolver la ecuación diferencial, obtenemos tres condiciones, bajo amortiguamiento críticamente amortiguado y sobreamortiguado dependiendo del valor de la resistencia. ¿Podría decirme más cómo una resistencia o un valor de resistencia puede remodelar esta onda sinusoidal a otra cosa?
subamortiguado es un gráfico en el que la amplitud de la onda sinusoidal se vuelve cada vez menor con respecto al tiempo y finalmente desaparece. eso tiene sentido porque en cada ciclo de corriente, la resistencia consume parte de la energía, pero no entiendo qué sucede en el caso de un caso crítico (R cuadrado = 4L / C) y sobreamortiguado (R> 4L / C) como en esos casos no se forma una onda sinusoidal.
@Alex He agregado una actualización a mi respuesta como resultado de sus comentarios recientes.
realmente muchas gracias @Farcher. solo una pregunta mas. cuando el voltaje a través del capacitor aumente, la corriente disminuirá y viceversa. cuando el voltaje a través del inductor aumente, la corriente también disminuirá y viceversa. en el circuito LC con fuente de CC cuando el voltaje a través del capacitor aumenta, el voltaje a través del inductor disminuye con respecto al tiempo. Entonces, ¿en qué punto la corriente alcanzará su valor máximo en el circuito a medida que el aumento de V (C) y V (L) causa una disminución de la corriente y cuando uno aumenta, el otro disminuye en ese momento?
Lo siento, te molesté mucho, pero esta es mi última pregunta al respecto. lamento molestarte te estare muy agradecido.
@Alex He trazado algunos gráficos de voltaje y corriente con la esperanza de que puedan responder a su pregunta, pero parece que no puedo cargarlos.
@Alex Sin resistencia en el circuito, la corriente es un máximo positivo, VL es cero y VC es +1. Luego, cuando la corriente es cero, VL es negativo máximo en -1 y VC es +2. Cuando la corriente máxima negativa, VL es cero y Vc es +1. Entonces, cuando VL pasa de 0 a -1 a 0, VC pasa de +1 a +2 a +1, siendo la suma del voltaje siempre +1.
@Farcher, ¿puede responder esta pregunta? No recibo ninguna respuesta satisfactoria al respecto. physics.stackexchange.com/questions/405075/lc-circuit-current

Como desea evitar las ecuaciones diferenciales, consideraré el llamado dominio fasorial , que en realidad no es más que la transformada de Fourier de las señales originales.

En el dominio fasorial, básicamente consideraremos valores complejos: voltajes complejos, resistencias complejas (que se denotan por Z y llamado impedancia): Esto es simplemente la conveniencia matemática, y siempre obtendremos los valores físicos reales al final.

Por el momento, olvídate de las partes activas del circuito y concéntrate en los elementos pasivos, es decir, olvídate del interruptor y la fuente de CC. Ahora veamos el comportamiento de los elementos restantes si tuviéramos que dar el voltaje complejo V ( t ) = mi i ω t :

  • Para el condensador, tenemos la ecuación. C d V d t = I , por lo tanto obtenemos

    I = i ω C mi i ω t
    lo que significa que la impedancia es
    Z = V I = 1 i ω C

  • Para el inductor, tenemos la ecuación. L d I d t = V , por lo tanto obtenemos

    i ω L I = mi i ω t
    lo que significa que la impedancia es
    Z = V I = i ω L

Dado que el capacitor y el inductor están en serie, la impedancia total es su suma (como en el caso de las resistencias), por lo que la impedancia de la parte pasiva del circuito es

Z = i ω L + 1 i ω C = 1 ω 2 L C i ω C

Esto significa que el voltaje a través del capacitor para una señal de CA sinusoidal sería

V condensador = V aporte Z condensador Z total = V aporte 1 i ω C 1 ω 2 L C i ω C
por lo tanto, la relación entre el voltaje de entrada y el voltaje de salida es
V producción V aporte = 1 1 ω 2 L C
donde tomamos el voltaje de salida como el voltaje del capacitor.

Uno puede notar inmediatamente que la relación iría al infinito si fuéramos a excitar el sistema con una fuente de CA de la frecuencia

ω = 1 L C
Esto se llama la frecuencia de resonancia. A esta frecuencia, el sistema acumula cada ciclo de energía entrante y aumenta su salida indefinidamente. En el mundo real, la existencia de resistencias en los sistemas y la avería de los equipos evita esta divergencia hasta el infinito.

Si ahora volvemos a nuestra pregunta, ahora simplemente necesitamos saber cómo podemos escribir nuestra excitación en términos de ondas sinusoidales puras de frecuencias constantes. Si sabemos esto, podemos examinar la salida del sistema porque estamos tratando con un sistema lineal: el efecto de la suma de las señales sinusoidales es igual a la suma de los efectos de las señales individuales.

Nuestra señal de entrada es simplemente una función escalonada, o la llamada función Heaviside-theta, que es cero antes t = 0 y constante después t = 0 donde elegimos t = 0 como la hora en que se enciende el interruptor. Ahora hay que descomponer la función Heaviside-theta en sinusoidales puras: me saltaré el cálculo aquí; es básicamente la transformación de Fourier de esta función. La respuesta es

V aporte ( F ) = mi ( i 2 π F + π 2 d ( F ) )
lo que significa que estamos creando una señal de entrada que se descompone de V aporte ( F ) para todas las frecuencias F . Ahora podemos encontrar la señal de salida para individuos F valores utilizando la proporción anterior:

V producción ( F ) = mi 1 4 π 2 F 2 L C ( i 2 π F + π 2 d ( F ) )
donde usamos el hecho de que la frecuencia natural ω es 2 π F .

Ahora podemos obtener nuestro resultado tomando la transformada inversa de Fourier, sumando efectivamente la contribución de todas las sinusoidales puras con diferentes ω valores: Nuevamente, esto se debe al hecho de que el sistema es lineal. Me saltaré los cálculos, pero puede usar fácilmente Mathematica para que lo haga por usted, como se puede ver en la imagen adjunta. El resultado es

V producción ( t ) = { mi ( 1 1 2 porque ( t 2 π C L ) ) t > 0 0 t < 0
lo cual es matemáticamente cierto pero con la condición límite incorrecta. Comento sobre esto más abajo. No obstante, nos muestra el comportamiento relevante.

Puede ver en la imagen que el voltaje simplemente sigue oscilando. Los gráficos reflejan tres hechos:

  • La energía sigue pasando de almacenarse en el capacitor a almacenarse en el inductor.
  • Como no hay resistencia, no hay efecto amortiguador, ¡así que las oscilaciones no mueren!
  • La forma de salida solo depende de las características del circuito pasivo; a saber, la inductancia del inductor y la capacitancia del condensador. El voltaje de entrada solo escala la forma de salida. Este es un concepto genérico: en ingeniería, esta característica se denomina respuesta de impulso, mientras que en física se conoce como la función de Green, aunque la respuesta de impulso es en realidad un caso específico de las funciones de Green más generales.

EDICIONES (Respuestas a los comentarios):

  • Hubo un error tipográfico antes donde escribí L C 4 en lugar de L C . Lo corregí arriba, ahora todo es dimensionalmente consistente.
  • El resultado respeta el valor límite en t = 0 ; sin embargo, este valor límite está mal definido. Sabemos que el voltaje de entrada es mi para t > 0 y 0 para t < 0 ; sin embargo, no sabemos qué es exactamente en t = 0 ya que encender un interruptor es un salto discontinuo. La opción matemática habitual es tomarlo como el valor medio, por lo tanto mi / 2 en t = 0 , esto también está en la definición de Heaviside theta en Mathematica, y es por eso que obtenemos mi / 2 para t = 0 en nuestra salida! Estaba completamente equivocado aquí, me di cuenta gracias a @Alfred
  • Si uno quiere definir el comportamiento del interruptor de manera diferente, puede repetir los cálculos en consecuencia (en realidad, usar las ecuaciones diferenciales sería mucho más fácil). La salida no tiene nada que ver con el interruptor sino con la condición límite elegida. Sin embargo, con la condición de contorno elegida, al final, la respuesta estática mi en la salida permanecería igual mientras que la respuesta de salto mi 2 porque ( t 2 π L C ) escalaría a mi porque ( t 2 π L C ) , por lo tanto tendríamos
    V producción = V condensador = mi ( 1 porque ( t 2 π L C ) )
    por eso
    I = C d V condensador d t = mi 2 π C L pecado ( t 2 π L C )

ingrese la descripción de la imagen aquí

Más comentarios

En mi opinión, el enfoque más simple para la pregunta es usar las ecuaciones diferenciales. Sin embargo, dado que OP nos pidió que evitemos esto, traté de explicarlo utilizando los hechos de que los sistemas eléctricos son sistemas lineales y que las funciones propias de los sistemas lineales son funciones exponenciales; eso es sinusoidales puras. Creo que esta es la aproximación conceptualmente más cercana al uso de ecuaciones diferenciales, aunque no igualmente rigurosa. Abordaré algunos problemas en esta parte con respecto al enfoque anterior.

En primer lugar, la transformada inversa de Fourier de nuestra salida,

V producción ( F ) = mi 1 4 π 2 F 2 L C ( i 2 π F + π 2 d ( F ) )
es en realidad
V producción = firmar ( t ) pecado 2 ( t 4 π C L ) + 1 2
que tiene el comportamiento como el siguiente diagrama:ingrese la descripción de la imagen aquí

Hay algunos comandos en orden:

  • La trama parece ingenuamente no causal, ya que hay una señal distinta de cero incluso antes de que se encienda el interruptor. Sin embargo, esa señal es en realidad la señal resonante que mencionamos anteriormente. Como discutimos allí, la resistencia efectiva (impedancia) para esta señal es cero para el circuito, por lo que esa señal nunca puede desaparecer en una solución genérica: ¡No tiene nada que ver con que el interruptor se encienda o se apague! El efecto del interruptor es simplemente cambiar la salida, de manera continua, lo cual es causal.
  • En el lenguaje de las ecuaciones diferenciales, la señal resonante es la solución homogénea que está presente en el sistema a menos que sea eliminada por una condición límite/inicial. El efecto del interruptor es la solución particular, que es simplemente el cambio de la señal de salida de manera causal.
  • En la parte de EDICIONES anterior, me confundí y dije incorrectamente que la razón por la cual el voltaje es 1 / 2 en t = 0 se debe a la convención de Heaviside-theta. Esto no es verdad. Creo que tiene que ver con el hecho de que se supone que las señales convergen a cero en menos infinito en la transformada de Fourier, por lo tanto, el valor medio del seno debe ser 0 para t < 0 , obligando a que la señal sea 1 / 2 en t = 0 permanecer continuo con el t > 0 parte. Si, en cambio, imponemos la condición de que debe ser 0 en t = 0 , obtenemos el resultado correcto para t > 0 , y señal irrelevante/no física para t < 0 .
  • Un método mucho mejor para hacer este cálculo es usar el dominio de Laplace en lugar del dominio de Fourier, donde consideramos la transformación de Laplace de 0 a inf . Esto nos permitiría implementar condiciones de contorno directamente en el dominio de Laplace y no obtendríamos la parte no física para t < 0 . Uno también puede hacer eso; Lo evité porque no busco un método riguroso o eficiente, sino uno conceptualmente fácil, y el espacio fasorial es más fácil de entender en mi opinión.

Para justificar las explicaciones anteriores, establezcamos las ecuaciones diferenciales:

I C = I yo d I C d t = d I yo d t + a L C d 2 V C d t 2 = V yo L + a L L C d 2 V C d t 2 = V i V C + a ( L C d 2 d t 2 + 1 ) V C = V i + a
dónde V i es el voltaje de entrada.

La solución homogénea es la de

( L C d 2 d t 2 + 1 ) V C = 0
que es simplemente
V C homogéneo = b porque ( t + C L C )
Esta es exactamente la misma señal que observamos para t < 0 en la trama de arriba.

El procedimiento correcto es en realidad usar la función de Green con la señal de entrada con las condiciones de contorno apropiadas: esto es lo que los ingenieros de EE llamarían respuesta de impulso. El dominio de Laplace es ideal para este trabajo en sistemas lineales. Mi explicación anterior, por otro lado, corresponde a encontrar una solución particular para V i = mi en cambio, combinándolo con una solución homogénea, imponiendo V mi ( t = 0 ) = 0 y desechando t < 0 parte.

La solución que das para V o tu t pag tu t ( t ) no puede ser correcto si este es el voltaje a través del capacitor. Como OP, la condición inicial para el voltaje del capacitor es v C ( 0 + ) = 0 .
@Alfred, edité la publicación para explicarlo.
Esta respuesta es conceptualmente defectuosa. El concepto de impedancia (la relación de los fasores de voltaje y corriente) asume una excitación sinusoidal. En el contexto de señales y sistemas, es bien sabido que la respuesta a una excitación general está dada por la convolución de la entrada X ( t ) con la respuesta al impulso del sistema h ( t ) , es decir, y ( t ) = X ( t ) h ( t ) . Para un sistema físico, h ( t ) debe ser causal , es decir, h ( t ) = 0 para t < 0 . Por el teorema de la convolución, Y ( F ) = X ( F ) H ( F ) . Pero, en lo anterior, el H ( F ) dada es no causal . El dominio de Laplace es mucho más apropiado aquí.
@Alfred, estoy totalmente de acuerdo en que la transformada de Laplace es conceptualmente mucho más apropiada: no por la causalidad, sino por el hecho de que tenemos una condición inicial. Con la transformación de Laplace, podríamos imponer la condición inicial directamente en el dominio transformado. Sin embargo, en mi opinión, la transformada de Fourier es conceptualmente más fácil de explicar y es suficiente (nuevamente, en mi opinión) en el caso actual.
@Alfred La señal de salida es ingenuamente no causal, ya que hay una parte distinta de cero para t <0. Esto se deduce de que con la transformada inversa de Fourier obtenemos la señal más genérica que incluye la señal resonante para t < 0 : ¡Esta señal es una solución homogénea de la ED correspondiente y siempre está allí a menos que las condiciones de contorno la eliminen! Así que sí, ingenuamente hay una señal distinta de cero para t<0 pero este no es el efecto del interruptor (por lo que no hay una relación causal) , es decir, no es una solución particular de DE. Podemos matarlo con condiciones de contorno: en el caso anterior, simplemente lo descarté centrándome en t > 0 parte solamente.

Ahora sabemos que la corriente en el inductor aumenta mientras que en un condensador la corriente disminuye con respecto al tiempo.

En general, esto no es cierto y ciertamente no puede ser cierto aquí, ya que el inductor y el capacitor están conectados en serie y, por lo tanto, tienen una corriente idéntica.

Si reemplaza el inductor con una resistencia (para formar un circuito RC en serie), puede concluir que la corriente del capacitor disminuye con el tiempo después de que se cierra el interruptor (suponiendo una condición inicial cero).

De manera similar, si reemplaza el capacitor con una resistencia (para formar un circuito RL en serie), puede concluir que la corriente del inductor aumenta con el tiempo después de que se cierra el interruptor (suponiendo una condición inicial cero).

Pero es un error aplicar estas conclusiones a un circuito LC en serie. Sin embargo, puedes hacerte una idea del gráfico de la corriente razonando cuidadosamente.

Primero, tenga en cuenta que, en todo momento después de que se cierre el interruptor, la suma instantánea del voltaje a través del inductor y el capacitor debe ser igual al voltaje de la batería. mi .

Segundo, reconozca que la corriente en serie inicialmente debe estar aumentando debido al voltaje mi a través del inductor (ya que el voltaje inicial a través del capacitor es cero). Entonces, razona de la siguiente manera:

A medida que aumenta el voltaje en el capacitor (debido a la corriente de carga), el voltaje en el inductor debe disminuir (para mantener la suma igual a mi ) y así la tasa de aumento de la corriente en serie disminuye , es decir, la corriente sigue aumentando pero no aumentando tan rápidamente .

En algún momento, el voltaje a través del capacitor alcanza mi y luego el voltaje a través del inductor es cero y, por lo tanto, la corriente en serie ha dejado de cambiar: la corriente en serie ha alcanzado su valor máximo y el capacitor se está cargando a su velocidad máxima.

El capacitor continúa cargándose (debido a la corriente) y, por lo tanto, el voltaje en el capacitor ahora excede mi que requiere que el voltaje del inductor se vuelva negativo (para mantener la suma igual a mi ). Debido a que el voltaje del inductor es negativo, la corriente en serie ahora debe disminuir .

El capacitor continúa cargándose pero no tan rápido ya que la corriente está disminuyendo y, eventualmente, la corriente disminuye a cero, el capacitor ha alcanzado su voltaje máximo y el voltaje del inductor es ahora su valor más negativo (para mantener la suma igual a mi ). Esto significa que la corriente está disminuyendo más rápidamente y, de hecho, la corriente disminuye hasta cero para volverse negativa.

La corriente negativa inicia la descarga del capacitor, el voltaje del capacitor comienza a disminuir y así el voltaje a través del inductor se vuelve menos negativo. A medida que el voltaje del condensador disminuye a mi , el voltaje del inductor una vez más es cero y la corriente en serie ya no cambia: la corriente en serie ha alcanzado su valor más negativo y el capacitor se descarga a su tasa máxima.

A estas alturas, supongo, puedes rastrear fácilmente el resto del ciclo. El capacitor continúa descargándose hasta que su voltaje es cero, la corriente en serie es cero y el voltaje del inductor es mi en cuyo punto el circuito ha vuelto al estado en t = 0 + y el ciclo se repite.

La corriente a través de un capacitor está determinada (limitada) por los componentes externos (e, indirectamente, por el voltaje en el capacitor) y aumentará, disminuirá o permanecerá en cero, dependiendo de lo que suceda en el resto del circuito.

Por ejemplo, si un capacitor está conectado a una batería, su corriente estaría determinada por la diferencia de voltaje entre la batería y el capacitor y la resistencia interna de la batería. A medida que se carga el capacitor, la diferencia de voltaje disminuye y, por lo tanto, la corriente (Vbat-Vcap)/r disminuye.

En su circuito, el capacitor está conectado en serie con el inductor que, como ha señalado, inicialmente se comportará como un circuito abierto, por lo que la corriente inicial a través del capacitor, determinada por el inductor, será cero.

A medida que aumenta la corriente a través del inductor (y el capacitor), inicialmente como di/dt=Vbat/L, el capacitor comenzará a cargarse. Esto reducirá la caída de tensión en el inductor y, en consecuencia, la tasa de crecimiento de la corriente, di/dt=(Vbat-Vcap)/L.

Entonces, la corriente en el circuito comenzará en cero y luego crecerá con una aceleración decreciente. Este es en realidad el comienzo de una onda sinusoidal con una frecuencia de 1/2π√(LC).